Menu Close

Category: None

Suppose-1-2-4-are-the-roots-of-the-equation-x-4-ax-2-bx-c-0-What-is-the-value-of-c-

Question Number 100339 by bobhans last updated on 26/Jun/20 $$\mathrm{Suppose}\:\mathrm{1}\:,\mathrm{2}\:,\mathrm{4}\:\mathrm{are}\:\mathrm{the}\:\mathrm{roots}\:\mathrm{of}\:\mathrm{the}\:\mathrm{equation} \\ $$$${x}^{\mathrm{4}} \:+{ax}^{\mathrm{2}} +{bx}−{c}\:=\:\mathrm{0}\:.\:{W}\mathrm{hat}\:\mathrm{is}\:\mathrm{the}\:\mathrm{value}\:\mathrm{of} \\ $$$${c}\:?\: \\ $$ Answered by bemath last updated on 26/Jun/20…

solve-for-x-4x-2-x-

Question Number 34792 by mondodotto@gmail.com last updated on 11/May/18 $$\boldsymbol{\mathrm{solve}}\:\boldsymbol{\mathrm{for}}\:\boldsymbol{{x}} \\ $$$$\mathrm{4}\boldsymbol{{x}}=\mathrm{2}^{\boldsymbol{{x}}} \\ $$ Answered by Joel579 last updated on 11/May/18 Commented by Joel579 last…

please-help-me-to-solve-this-xln3-e-3yln3-0-lnx-2lny-1-

Question Number 100317 by pticantor last updated on 26/Jun/20 $$\boldsymbol{{please}}\:\boldsymbol{{help}}\:\boldsymbol{{me}}\:\boldsymbol{{to}}\:\boldsymbol{{solve}}\:\boldsymbol{{this}}! \\ $$$$ \\ $$$$\:\:\:\:\:\begin{cases}{\boldsymbol{{xln}}\mathrm{3}−\boldsymbol{{e}}^{\mathrm{3}\boldsymbol{{yln}}\mathrm{3}} =\mathrm{0}}\\{\boldsymbol{{lnx}}−\mathrm{2}\boldsymbol{{lny}}=\mathrm{1}}\end{cases} \\ $$$$ \\ $$ Answered by MJS last updated on…

Question-100311

Question Number 100311 by mathocean1 last updated on 26/Jun/20 Answered by mathmax by abdo last updated on 26/Jun/20 $$\mathrm{M}\left(\mathrm{f},\mathrm{B}\right)\:=\begin{pmatrix}{\mathrm{1}\:\:\:\:\:\:\mathrm{2}}\\{\mathrm{1}\:\:\:\:\:\:\:\mathrm{0}}\end{pmatrix}\:\:\Rightarrow\mathrm{f}\left(\mathrm{i}\right)\:=\mathrm{i}+\mathrm{j}\:\mathrm{and}\:\mathrm{f}\left(\mathrm{j}\right)\:=\mathrm{2i} \\ $$$$\mathrm{g}\:=\mathrm{f}+\mathrm{id}_{\mathrm{E}} \:\:\:\mathrm{and}\:\mathrm{h}\:=\mathrm{f}−\mathrm{2id}_{\mathrm{E}} \:\Rightarrow\mathrm{g}\left(\mathrm{u}\right)\:=\mathrm{f}\left(\mathrm{u}\right)+\mathrm{u}\:\:\mathrm{and}\:\mathrm{h}\left(\mathrm{u}\right)=\mathrm{f}\left(\mathrm{u}\right)−\mathrm{2u}\:\Rightarrow \\ $$$$\mathrm{g}\left(\mathrm{i}\right)\:=\mathrm{f}\left(\mathrm{i}\right)+\mathrm{i}\:=\mathrm{2i}+\mathrm{j}\:\:\mathrm{and}\:\:\mathrm{g}\left(\mathrm{j}\right)\:=\mathrm{f}\left(\mathrm{j}\right)+\mathrm{j}\:=\mathrm{2i}+\mathrm{j}\:\Rightarrow…

Version-2-085-is-now-available-on-playstore-Please-update-

Question Number 100310 by Tinku Tara last updated on 26/Jun/20 $$\mathrm{Version}\:\mathrm{2}.\mathrm{085}\:\mathrm{is}\:\mathrm{now}\:\mathrm{available}\:\mathrm{on} \\ $$$$\mathrm{playstore}.\:\mathrm{Please}\:\mathrm{update}. \\ $$ Commented by Tinku Tara last updated on 26/Jun/20 $$\mathrm{You}\:\mathrm{have}\:\mathrm{made}\:\mathrm{this}\:\mathrm{request}\:\mathrm{before}. \\…

Question-100304

Question Number 100304 by Algoritm last updated on 26/Jun/20 Commented by PRITHWISH SEN 2 last updated on 26/Jun/20 $$\mathrm{t}_{\mathrm{n}} =\frac{\mathrm{n}}{\left(\mathrm{n}+\mathrm{1}\right)!}\:=\:\frac{\mathrm{1}}{\mathrm{n}!}\:−\frac{\mathrm{1}}{\left(\mathrm{n}+\mathrm{1}\right)!}\: \\ $$$$\mathrm{and}\:\mathrm{it}\:\mathrm{is}\:\mathrm{a}\:\mathrm{telescopic}\:\mathrm{series} \\ $$ Commented…

Question-100290

Question Number 100290 by Harasanemanabrandah last updated on 26/Jun/20 Commented by Rasheed.Sindhi last updated on 26/Jun/20 $$\:^{\bullet} {If}\:\:{n}\:{is}\:{multiple}\:{of}\:\mathrm{3}\:\:{then}\:\mathrm{3}^{{n}} +{n}^{\mathrm{3}} \: \\ $$$${is}\:{obviously}\:\:{composite}\:\left(\mathrm{3}\:\mid\:\mathrm{3}^{{n}} +{n}^{\mathrm{3}} \right) \\…

Question-100285

Question Number 100285 by DGmichael last updated on 26/Jun/20 Answered by mathmax by abdo last updated on 26/Jun/20 $$\mathrm{p}\:=\frac{\mathrm{1}−\mathrm{a}^{\mathrm{2}^{\mathrm{n}+\mathrm{1}} } }{\mathrm{1}−\mathrm{a}^{\mathrm{2}} }\:\:\:\:\mathrm{we}\:\mathrm{can}\:\mathrm{prove}\:\mathrm{this}\:\mathrm{by}\:\mathrm{recurrence}\:\:\left(\mathrm{a}^{\mathrm{2}} \:\neq\mathrm{1}\right) \\ $$…